The Olympiad Problem That DEFEATED Britain's Best...

แชร์
ฝัง
  • เผยแพร่เมื่อ 26 ต.ค. 2024

ความคิดเห็น • 63

  • @OscgrMaths
    @OscgrMaths  3 หลายเดือนก่อน +4

    There are lots of approaches to this question so feel free to share here!

  • @rhyslewis9057
    @rhyslewis9057 3 หลายเดือนก่อน +48

    Just commenting to say I'm actually the one that came up with this problem and I'm really touched by all the comments about it!

    • @OscgrMaths
      @OscgrMaths  3 หลายเดือนก่อน +14

      @@rhyslewis9057 Wow! It's an absolutely incredible problem and I've loved thinking about it - thank you. I hope I did it justice.

    • @MyOneFiftiethOfADollar
      @MyOneFiftiethOfADollar หลายเดือนก่อน +1

      Just commenting to say How do we know you are telling the truth?

  • @maths_505
    @maths_505 3 หลายเดือนก่อน +8

    I honestly don't watch this kind of math but your teaching style encourages me to watch right till the end. Excellent work once again.

    • @OscgrMaths
      @OscgrMaths  3 หลายเดือนก่อน +4

      Bro thanks so much! I really appreciate that.

    • @Player_is_I
      @Player_is_I 2 หลายเดือนก่อน +1

      Maths 505 is here! Love your vids❤

    • @maths_505
      @maths_505 2 หลายเดือนก่อน +3

      @@Player_is_I thanks mate

  • @josephbrisendine2422
    @josephbrisendine2422 3 หลายเดือนก่อน +2

    Commenting to help you with the algorithm. This was a good one and I hope one day you have as many subscribers as Michael Penn; keep it up!

    • @OscgrMaths
      @OscgrMaths  3 หลายเดือนก่อน

      @@josephbrisendine2422 Thanks so much! Very kind of you.

  • @riot1133
    @riot1133 3 หลายเดือนก่อน +8

    i can see why this problem defeated so many olympiad solvers, couldn't imagine coming up with the solution myself. you explained it very well though, enjoyed solving this with you

    • @OscgrMaths
      @OscgrMaths  3 หลายเดือนก่อน +1

      @@riot1133 Thanks so much! It is very tough but a really interesting question all the same.

  • @anas.aldadi
    @anas.aldadi 3 หลายเดือนก่อน +4

    What i like about your channel is that it opens my mind to think differently literally! i pause the vid to try and solve it myself but I find you solve it in another way!

    • @OscgrMaths
      @OscgrMaths  3 หลายเดือนก่อน +1

      @@anas.aldadi Thanks so much! This is especially true for olympiad problems which could have lots of different constructions to reach the same result.

  • @حسينالقطري-ب8ص
    @حسينالقطري-ب8ص 3 หลายเดือนก่อน +2

    Interesting problem and nice and systematic explanation!
    Thank you very much for making such a good video ❤

    • @OscgrMaths
      @OscgrMaths  3 หลายเดือนก่อน

      @@حسينالقطري-ب8ص Thank you!

  • @ShubhamHooda-fz7ci
    @ShubhamHooda-fz7ci 2 หลายเดือนก่อน +1

    Try some "JEE advanced"Questions, specially that probability stuff from there

  • @amarstatovci2505
    @amarstatovci2505 2 หลายเดือนก่อน +1

    as a person who just finished my first year of ChemE, I have no idea what 90% of that was but I still watched it

  • @decaf3821
    @decaf3821 3 หลายเดือนก่อน

    Can you do a video on analytic continuation? Especially with the zeta function in the complex plain. I can’t wrap my head around, nor find any good videos on how the analytically-continued zeta function is derived

  • @niom-nx7kb
    @niom-nx7kb 3 หลายเดือนก่อน +2

    very clear answer, and this is also one of my favourite questions as it mainly involves prime factors and regular factors of positive integers! but I have a question that I dont understand. when you proved that f(a^n * b) is divisible by N, but dont we also have to prove that f(a1 ^ k1 * a2 ^ k2 * ... as ^ ks) is divisible by N in order to answer the question?

    • @OscgrMaths
      @OscgrMaths  3 หลายเดือนก่อน +2

      @@niom-nx7kb Hey! All we have to do is show that there exists an n such that f(n) will be a multiple of N for all positive integers N and we've already achieved this with just the case of f(a^k b).
      Choose any integer you like - let's do 30 for example but it works with any greater than 2. if N = 30, i'll choose n= a^28 * b
      f(a^28 * b) = (28+2)*2^27
      = 30 * 2^27
      And so it's a multiple of 30. Choosing n= a^(N-2)*b
      will yield a multiple of N for any possible case larger than N=2 (which we already dealt with) and so it's been proven!

    • @niom-nx7kb
      @niom-nx7kb 3 หลายเดือนก่อน +2

      @@OscgrMaths oh now I get it

  • @aleksandervadla9881
    @aleksandervadla9881 2 หลายเดือนก่อน +1

    Did you compete in the BMO?

  • @trueboxguy5421
    @trueboxguy5421 3 หลายเดือนก่อน +2

    I actually attempted this problem in the BMO! I believe I got 9/10. My solution was pretty similar to yours (I have it but it's difficult to share on TH-cam as it doesn't allow links to images).
    I can share the feedback I got for this kind of solution (which I believe is essentially what you did, but I made some arithmetic errors): Nice solution. Check for N=2 case carefully: you’re right to pay attention, but f(6)=3. f(4) is a better way to deal with it.

    • @OscgrMaths
      @OscgrMaths  3 หลายเดือนก่อน +1

      At the end I mentioned it, f(2^2) is the best way to go about the special case! Thanks so much the comment and that's great that you sat it in the real thing! Incredible to have gotten 9/10 under those conditions - which other questions did you attempt?

    • @trueboxguy5421
      @trueboxguy5421 3 หลายเดือนก่อน +2

      @@OscgrMaths I attempted questions 1, 2, 3 and 5. (Also if you're sitting any olympiads/STEP in the future etc: good luck! This olympiad was basically the only exam I ever had a good performance in lol)

    • @OscgrMaths
      @OscgrMaths  3 หลายเดือนก่อน +1

      @@trueboxguy5421 I might do a video on question 6... I've not seen an olympiad problem like that before, I'm sure it confused most people in the exam!

    • @trueboxguy5421
      @trueboxguy5421 3 หลายเดือนก่อน +1

      ​@@OscgrMaths I'll subscribe to you (on my other account) so I can watch it if it comes out!

    • @OscgrMaths
      @OscgrMaths  3 หลายเดือนก่อน

      @@trueboxguy5421 Okay great!

  • @expectus4673
    @expectus4673 3 หลายเดือนก่อน +1

    Dont know how I ended up here but glad I did, I somewhat understood the proof but noway I could have thought of it lol

    • @OscgrMaths
      @OscgrMaths  3 หลายเดือนก่อน +1

      @@expectus4673 I think that's the hardest thing about this question - finding a good starting place or plan of attack since it seems so bleak at first. Glad you enjoyed and thanks for the comment!

  • @Player_is_I
    @Player_is_I 2 หลายเดือนก่อน +2

    Great vid❤

    • @OscgrMaths
      @OscgrMaths  2 หลายเดือนก่อน +1

      @@Player_is_I Thank you so much! I really appreciate it.

  • @heraklisxa6005
    @heraklisxa6005 3 หลายเดือนก่อน

    This problem is similar to the problem of finding the positive integers n such that n | φ(n) where φ(n) Euler's totient function, which are of the form: n= (2^m)*(3^l).
    Instead, in this problem we are counting all the lists that we can create from the set of all the divisors of n.
    So aren't we counting all the permutations we can have for the prime factors of n? So if the canonical prime factorization of n = (p_{1}^a_{1})*...*(p_{k}^a_{k})
    Then what we are counting is a how many lists L are there such that: L = {p_{i}^a_{i} divides p_{i+1}^a_{i+1} | i in I} for I subset of I' = {1,2,...,k}
    Or am I completely wrong?

  • @mohammedgolfa3936
    @mohammedgolfa3936 3 หลายเดือนก่อน

    I just want to ask about at 10:20 using his way of swapping the upper row to the lower row of numbers, wouldn’t it led to double counting of the same sequence if the generalise it by 2^k-1 for swapping each time

  • @wjeksababakqabzzhzaab873
    @wjeksababakqabzzhzaab873 3 หลายเดือนก่อน +1

    PLEASE DO MORE BMO Qs!!!!!!

    • @OscgrMaths
      @OscgrMaths  3 หลายเดือนก่อน

      @@wjeksababakqabzzhzaab873 Will do!! Glad you enjoyed it!!

  • @uggupuggu
    @uggupuggu หลายเดือนก่อน +1

    I dont get the question? isnt that always true?
    N = 1 will always divide f(n) lol

    • @OscgrMaths
      @OscgrMaths  หลายเดือนก่อน

      @@uggupuggu Ah I see what you mean! You have to show that for any N, there will be a value of f(n) that N divides. Hope this helps!!

  • @DARKEST-2114
    @DARKEST-2114 3 หลายเดือนก่อน +3

    Let me ask you a very easy question if sec A + tan A= z then what will sec A - tan A = ??

    • @OscgrMaths
      @OscgrMaths  3 หลายเดือนก่อน +7

      1/z? since sec^2A - tan^2 A = 1 = (secA+tanA)(secA-tanA)

    • @DARKEST-2114
      @DARKEST-2114 3 หลายเดือนก่อน +2

      That's great why do you have put ? There be confident in your answer . If you are not confident in your answer that means you it's the answer is not correct as per you . My maths always say be confident about your answer

    • @MyOneFiftiethOfADollar
      @MyOneFiftiethOfADollar หลายเดือนก่อน

      Let me tell you to PAY A TUTOR rather than ask an off topic question.

  • @Lagrange-x4u
    @Lagrange-x4u 2 หลายเดือนก่อน +1

    Why you deleted the jee video??

    • @OscgrMaths
      @OscgrMaths  2 หลายเดือนก่อน +1

      @@Lagrange-x4u Hey, thanks for asking! I was really sad to but it was just because there was one part where I felt my explanation could have been better and I didn't want to confuse any viewers since that's the opposite of what I want to do with this channel!

  • @theupson
    @theupson 2 หลายเดือนก่อน

    the obvious recurrence for f(n) is on its face a trainwreck for trying to factor in any way, so almost right away we are reduced to trying to brute force a formula for f(n) in special cases, imo a disappointing line to have to take. all prime factors identical? too simple. all prime factors distinct? too complicated. FA til you FO, i guess.
    at first reading the problem is one of those olympiad immortals, but it kind of falls flat.

  • @ytkerfuffles6429
    @ytkerfuffles6429 2 หลายเดือนก่อน

    i proved this by induction on n=3x2^k when i sat this paper (:

    • @OscgrMaths
      @OscgrMaths  2 หลายเดือนก่อน

      @@ytkerfuffles6429 Nice! How did you find the other questions?

    • @ytkerfuffles6429
      @ytkerfuffles6429 2 หลายเดือนก่อน

      @@OscgrMaths i did question 1 because it wasnt that hard but the rest of them i did after because i didnt have time time

    • @OscgrMaths
      @OscgrMaths  2 หลายเดือนก่อน

      @@ytkerfuffles6429 That's fair! I thought it was very tricky - particularly question 6. Have you sat any other BMO papers? If so do you have any favourites? I'd love to go through another question on the channel.

    • @ytkerfuffles6429
      @ytkerfuffles6429 2 หลายเดือนก่อน

      @@OscgrMaths i sat this years one and i got 47/60 and 20/40 in the bmo2

    • @OscgrMaths
      @OscgrMaths  2 หลายเดือนก่อน

      @@ytkerfuffles6429 WOAH! Incredible. Huge congrats! Which was your favourite from this year? (including BMO 2)

  • @alexs3243
    @alexs3243 3 หลายเดือนก่อน

    The question asks: given any N, find n such that N | f(n)
    I think u only showed this for N of the form b*a^k where a,b are primes. This does not cover all integers! E.g. N=2*3*5

    • @OscgrMaths
      @OscgrMaths  3 หลายเดือนก่อน +1

      @@alexs3243 It does! In your example when N = 2*3*5 = 30, I would choose k = N-2 which is 28 and so n = a^28 b
      f(a^28 b) = (28+2)2^27
      (by our conjecture that we proved)
      = 30*2^27
      this is clearly divisible by 30. It works this way for any integer! This means whatever you choose N as there exists an n (n= a^N-2 b) that satisfies the condition.

    • @alexs3243
      @alexs3243 3 หลายเดือนก่อน +3

      @@OscgrMaths My mistake, thanks for clarifying :)

    • @OscgrMaths
      @OscgrMaths  3 หลายเดือนก่อน +1

      @@alexs3243 No problem! Thanks for commenting with questions, I'm happy to have helped.

  • @comdo777
    @comdo777 3 หลายเดือนก่อน +1

    ASNWER=1(SIN-COS)(SIN+COS) ISIT